Sunteți pe pagina 1din 4

SOA/CAS COURSE 2 - INTEREST THEORY - QUIZ 2

General Annuities and Fund Return Measurement


S. Broverman, 2003 1. A loan of amount 1000 is repaid with 12 annual payments of 100 each starting one year after the loan is made. The annual effective interest rate is 3.5% for the first four years. Find the annual effective interest rate for the final 8 years. A. 2.12% B. 2.15% C. 2.18% D. 2.21% E. 2.24%

2. At annual effective rate of interest , a perpetuity paying 25 every 3 months has a present value of X at the time of the first payment. At the same interest rate, a perpetuity paying Y every month also has a present value X at the time of the first payment. Then Y is equal to which of the following (all rates are equivalent to annual effective interest rate )?
 A.   
B.      C.    

D.   

 E.  

3. A loan can be paid in full, at the same positive rate of interest, by either 6 monthly payments of 209.56 each (starting one month after the loan is made) or 12 monthly payments of 108.99 each (also starting one month after the loan is made). Find the corresponding payment for an 18 month repayment. A. 75.54 B. 75.90 C. 76.26 D. 76.72 E. 77.08

4. Bill deposits money into a bank account at the end of each year. Bill's deposit in year ! is equal to ! ! ~    The bank credits interest at an annual effective rate of . The amount of interest earned in Bill's account during the 11th year is equal to 500. Calculate . A. 7.25% B. 7.75% C. 8.00% D. 8.25% E. 8.75%

5. An investor pays 7 for an annuity which provides payments of 100 at the beginning of each month for 10 years. These payments are invested at a nominal annual interest rate of 12% convertible monthly. Monthly interest payments are reinvested at a nominal annual interest rate of 6% convertible monthly. The annual yield rate over the 10 year period is 8% effective. Calculate 7 . A. 9600 B. 9650 C. 9700 D. 9750 E. 9800

6. You are given an annuity-immediate paying 10 for 10 years, then decreasing by one per year for nine years and paying one per year thereafter, forever. The annual effective rate of interest is 4%. Calculate the present value of this annuity. A. 119 B. 121 C. 123 D. 125 E. 127

7. A perpetuity with annual payments is payable beginning 10 years from now. The first payment is 50. Each annual payment thereafter is increased by 10 until a payment of 150 is reached. Subsequent payments remain level at 150. This perpetuity is purchased by means of 10 annual premiums with the first premium of 7 due immediately. Each premium after the first is 105% of the preceding one. The annual effective interest rates are 5% during the first 9 years and 3% thereafter. Calculate 7 . A. 281 B. 286 C. 291 D. 296 E. 301

8. A stock is currently selling at 80 per share to yield 10% per annum compounded semi-annually. The stock is expected to pay dividends at the end of each year forever. The next dividend (payable one year from now) is 2 and is expected to increase at a rate of X% per year. Calculate X. A. 7.00 B. 7.25 C. 7.50 D. 7.75 E. 8.00

9. You are given the following information about an investment account: Jan. 1, 1999 Mar. 1, 1999 Apr. 1, 1999 T, 1999 108 102 118 Jan. 1, 2000 130

Account Value 100 (Before deposit or withdrawal) Deposit 20 X Withdrawal 12 The time-weighted yield rate for 1999 is 13.61%, and the dollar-weighted yield rate is 12.04%. Calculate T. A. May 1 B. July 1 C. September 1 D. October 1 E. November 1

10. On January 1, an investment account is worth 300,000. 4 months later, the value has increased to 315,000 and 15,000 is withdrawn. 4 months prior to the end of the year, the account is again worth 315,000 and 15,000 is withdrawn. On December 31, the account is worth 315,000. The annual effective yield rate, using the dollar-weighted method, is 16%. Calculate 4 . A. 2.00 B. 2.25 C. 2.50 D. 2.75 E. 3.00 2

SOA/CAS COURSE 2 - INTEREST THEORY - TEST 2 SOLUTIONS


S. Broverman, 2002
 1.  ~ O b # h  O S  O ~ 
 . Using the unknown interest function on the

calculator, we get  ~ % .

Answer: D

2. Suppose that  is the 3-month interest rate and  is the 1-month interest rate equivalent to .  @  Then ? ~  BO ~   ~  , where  ~ b is the 3-month discount rate, and  ~ b is the
 

1-month discount rate. Since  ~  Answer: C


O

and  ~  , we have




  

~  , @ ~   

@ 



3. 

O ~  O S  ~ 
. Since  ~  b #
, it follows that  ~  
O
O #
~  (interest is at a monthly rate). With monthly payment 2 for an 18-month repayment, we have 2 O ~ 

O so that 2~


O  O

O


c#
 c# 


c#
c#


c  c  

~  .

Answer: A

4. Bill's deposits form an increasing annuity. For the first 10 years, the annuity is a 10 year increasing annuity whose first deposit is 100 and subsequent deposits increase by 100 each year. The balance in the account at the end of the 10th year is 0 O . The interest earned in the 11th year is the interest, at rate , on the balance at the end of 10 years. The interest in the 11th year is O c 0 O h  ~ >  ? h  ~  O c  , which we are told is 500. Therefore, O ~  , and using the unknown interest function on the calculator gives us  ~   . Answer: A 5. The balances in the primary account are 100 at start of the first month of the first year, 200 at the start of the 2nd month of the first year, . . . , 12,000 at the start of the 12th month of the 10th year. The interest generated from that primary account at an interest rate of 1% per month will be 1 at the end of the first month of the first year, 2 at the end of the 2nd month of the first year, . . . , 120 at the end of the 12th month of the 10th year. These interest payments form a 120 payment (monthly) increasing annuity and are reinvested at .5% per month. The accumulated value of the reinvested interest is 0 O ~  . At the end of 10 years, the investor has a total amount of   b  ~   . To say that the annual yield rate over the 10 year period is 8% effective means that if 7 had been invested at 8% effective, it would have accumulated to 2,  . . Therefore, 7   ~   , so that 7 ~
. Answer: C

6. The annuity can be broken into 3 parts: 10 payments of 10 each, followed by 9 payments decreasing by 1 per year from 9 to 1 (from time 11 to 19), followed by a perpetuity of 1 per year (from time 20 on). The total of the present values of the three parts is
   O b # + O b # BO ~  c#  b #  b #  ~   .


c O

Answer: A

7. The premiums are paid from the beginning of the first year (time 0) to the beginning of the 10th year (time 9). The premiums form an annuity with geometrically increasing payments, with each payment  ~  b  times as large as the previous payment. Since the interest rate up to time 9 (the end of the first 9 years) is 5%, and is equal to , the accumulated value of the premiums at the time of the 10th premium (which occurs at time 9) is 7  . Time 9 is one year before the payment in the perpetuity. The perpetuity has payments of 50, 60, 70, . . . , 150, 150, 150, . . . This perpetuity can be expressed as a level perpetuity of 150, 150, 150, . . . minus the 10 payment decreasing annuity with payments 100, 90, 80, . . . , 10 . From time 9 on, the annual effective interest rate is 3%. The present value of the perpetuity at time 9 is  BO c +O ~   . Since the perpetuity is to be paid for by the premiums, we have 7  ~   , and then 7 ~    . Answer: C

8. The annual effective rate of interest is   c  ~  . The stock pays annual dividends that increase geometrically by  b ? ~  b  per year. The theoretical price of the stock is    ~ c ~  c? , so that ? ~ . Answer: D 9. From Jan 1 to Mar 1, the balance grows from 100 to 108; after the withdrawal of 12 on Mar 1, the balance is 96 and grows to 102 on Apr 1; after the deposit of 20 on Apr 1 the balance is 122 and reduces to 118 at time T; after the deposit of X at time T the balance is 118 b X and grows to 130 on

   Jan 1, 2000. The time-weighted yield is  


  b? c  , which we are told is 
 . It follows that ? ~ . The withdrawal of 12 occurs at time 
during the year, the deposit of 20

occurs at time   , and the deposit of ? ~ occurs at time ; . return  is dollar-weighted return is  ~ . deposit of ? ~ occurs at time .67 year). Answer: C

The equation for the dollar-weighted

  b  c  b
 b  b   b  b  c ;  ~  . We are given that the

From the dollar-weighted equation, we get ; ~


. The (  of the year) in the year, which is Sep 1 (8 months into the

10. The first withdrawal of 15,000 occurs 4 months after the start of the year (or with  c 4  years end of the year. The equation for the dollar-weighted return is 4  
c   b 
 c 4  c   b 
 ~   . Solving for 4 results in 4 ~  . Answer: E

remaining until the end of the year), and the second withdrawal of 15,000 occurs 4  years before the

S-ar putea să vă placă și